Double integrals: cartesian to polar coordinates

Click For Summary
To convert the given Cartesian integral to polar coordinates, the transformation equations x = r cos(θ) and y = r sin(θ) are used. The bounds for θ are correctly identified as π/4 to π/2, while the bounds for r depend on θ and can be determined by the geometry of the area of integration, which is a right triangle. Specifically, the projection r sin(θ) remains constant, leading to the relationship where r is defined in terms of θ as r = 6/cos(θ). The final evaluation of the integral should yield the answer of 36, as stated in the textbook. Understanding the relationship between r and θ is crucial for setting the correct limits for the polar integral.
mmont012
Messages
39
Reaction score
0

Homework Statement


Change the Cartesian integral into an equivalent polar integral and then evaluate.

Homework Equations


x=rcosθ
y=rsinθ

upload_2015-11-27_1-53-51.png

I have:
∫∫r2cosθ dr dθ

The bounds for theta would be from π/4 to π/2, but what would the bounds for r be?

I only need help figuring out the bounds, not with the evaluating.

The answer for the problem is 36 (or so says the back of the textbook).
 
Physics news on Phys.org
mmont012 said:
but what would the bounds for r be?
The area of integration forms a right triangle subtending angle from 45 to 90 degree, so the limit for r would be a function of ##\theta##. For a hint, as you sweep the triangle in between those two limiting angles, the projection ##r \sin \theta## is constant.
 
  • Like
Likes mmont012
For every theta, a line from the origin, making angle theta with the y-axis, to the line y= 6 is the hypotenuse of a right triangle with one leg of length 6. cos(\theta)= \frac{6}{h}.
 
  • Like
Likes mmont012
Thank you both!
 
Question: A clock's minute hand has length 4 and its hour hand has length 3. What is the distance between the tips at the moment when it is increasing most rapidly?(Putnam Exam Question) Answer: Making assumption that both the hands moves at constant angular velocities, the answer is ## \sqrt{7} .## But don't you think this assumption is somewhat doubtful and wrong?

Similar threads

  • · Replies 24 ·
Replies
24
Views
3K
  • · Replies 3 ·
Replies
3
Views
3K
  • · Replies 19 ·
Replies
19
Views
2K
  • · Replies 12 ·
Replies
12
Views
2K
  • · Replies 11 ·
Replies
11
Views
3K
  • · Replies 5 ·
Replies
5
Views
2K
  • · Replies 3 ·
Replies
3
Views
2K
  • · Replies 2 ·
Replies
2
Views
1K
Replies
3
Views
2K
  • · Replies 14 ·
Replies
14
Views
3K